Minimum value Probability Question

Click For Summary
To determine the minimum number of transmissions (n) required for a message to be received with at least a 95% probability, given a success probability (p) of 0.8, the equation 0.8 + 0.8 * ∑(1-0.8)^j must be satisfied. The initial attempts show that the first two transmissions yield a combined probability of 0.96, which exceeds the 0.95 threshold. However, confusion arises in setting up the summation correctly for further attempts. The discussion highlights the challenges of using LaTeX for mathematical expressions, emphasizing the need for clarity in notation. Ultimately, the solution revolves around correctly calculating the cumulative probabilities of multiple attempts.
asd1249jf

Homework Statement


When a conventional apging system transmits a message, the probability that the message will be received by the pager it is sent to is p. To be confident that a message is received at least once, a system transmits the message n times.

Assuming p = 0.8, what is the minimum value of n that produces a probability of 0.95 of receiving the message at least once?


The Attempt at a Solution


Ok, so the probability of message being transmitted per attempt is

First Attempt : 0.8
Second Attempt : 0.8(1-0.8)
Third Attempt : 0.8(1-0.8)^2

and so on.

So we could set up an equation

0.8 + \sum_{j=1}^\n \nnnnnnnnnnnnnnnn0.8(1-0.8)^j = 0.95

0.8 * \sum_{j=1}^\n \((1-0.8)^j = 0.15

\sum_{j=1}^\n \((1-0.8)^j = 0.1875

And I'm stuck here. There is no possible value for n for this summation to be satisfied (n needs to be an integer equal or greater than 1). Can anybody help?
 
Last edited by a moderator:
Physics news on Phys.org
Ok, it's situation like these latex seriously does a commendable job of pissing me off. How would you write the summation of index starting from j = 1 until n?
 
The probability the message is received on the first try is 0.8. The probability the messages is received on the second try given that the first one failed is (as you say) 0.8*(1-0.8)=0.16. The sum of the two is 0.96. Aren't you done?
 
l46kok said:
Ok, it's situation like these latex seriously does a commendable job of pissing me off. How would you write the summation of index starting from j = 1 until n?

The same way you did in your last latex block. \sum_{j=1}^n whatever. \sum_{j=1}^n a_j.
 
Last edited:
Question: A clock's minute hand has length 4 and its hour hand has length 3. What is the distance between the tips at the moment when it is increasing most rapidly?(Putnam Exam Question) Answer: Making assumption that both the hands moves at constant angular velocities, the answer is ## \sqrt{7} .## But don't you think this assumption is somewhat doubtful and wrong?

Similar threads

  • · Replies 2 ·
Replies
2
Views
2K
Replies
31
Views
6K
Replies
1
Views
2K
  • · Replies 1 ·
Replies
1
Views
2K
  • · Replies 1 ·
Replies
1
Views
2K
Replies
6
Views
2K
  • · Replies 29 ·
Replies
29
Views
4K
  • · Replies 17 ·
Replies
17
Views
2K
Replies
8
Views
2K
  • · Replies 3 ·
Replies
3
Views
1K